LSAT and Law School Admissions Forum

Get expert LSAT preparation and law school admissions advice from PowerScore Test Preparation.

 Administrator
PowerScore Staff
  • PowerScore Staff
  • Posts: 8917
  • Joined: Feb 02, 2011
|
#73470
Complete Question Explanation
(The complete setup for this game can be found here: https://forum.powerscore.com/lsat/viewtopic.php?t=3901)

The correct answer choice is (B)

If a rock group performs at 10, then from the third rule the other group performing at 10 must also be a rock group. This immediately affects Q, because according to the last rule Q must perform after a folk group. Thus, Q cannot perform at 12 (or at 6 from that same rule). Therefore, it appears that Q must perform at 8 or 10. However, Q cannot perform at 8 due to the following reasoning:
  • If Q, a rock group, performs at 8, then according to the last rule the group that performs at 6 must be a folk group. P, which is a rock group, therefore cannot perform at 6 and must instead perform at 12. However, this presents a problem because only one rock group remains unassigned—group T—yet two rock groups are still needed to play at 10. Thus, if Q performs at 8 under the conditions of this question, no viable solution can be produced. Hence, Q cannot perform at 8, and Q must instead perform at 10.
If you did not see this reasoning (and it is very tough to do so abstractly), once you reached the point of knowing that Q must perform at 8 or 10, why not try some hypotheticals to help eliminate answer choices? Very quickly you would discover that Q could not perform at 10, and that would lead you directly to the correct answer.

Now that we have established that Q performs at 10, from the last rule we can deduce that a folk group must play at 8, and hence answer choice (B) is correct.

Combining all of the information above leads to the following final setup to this tough game:
D05_Game_#4_setup_diagram_full_a.png
D05_Game_#4_setup_diagram_full_b.png
Given all of the rules above, you might expect that this game has some serious limitations, and indeed there are only four basic directions the game can take where the performers can be determined at each time (but the exact stage for all groups—at 8 and 10 in particular—cannot be determined). However, for most students these are very tough to identify in the time allotted, and thus we have chosen to display just the inferences above.
You do not have the required permissions to view the files attached to this post.
 rwraulynaitis
  • Posts: 27
  • Joined: May 06, 2020
|
#76784
I took a very different approach on this question that allowed me to answer it very quickly, but I wanted to check to see if it was a sound strategy.

With the local condition of a rock group performing at 10, I knew that two rock groups would then be performing at 10, leaving only one rock group still to be placed elsewhere. Then, because two groups must perform at 8, and there is only one rock group remaining, I reasoned that there must also be at least one folk group who plays at 8, leading me to answer choice (B).

Was this a good strategy, potentially one I could use in similar questions?
 Adam Tyson
PowerScore Staff
  • PowerScore Staff
  • Posts: 5153
  • Joined: Apr 14, 2011
|
#76955
That's fantastic, rwraulynaitis! Fast, easy, and accurate, a brilliant inference. Frankly, I wish I had thought of that myself! My approach was a little slower than yours, but a bit less involved than the one in our official explanation. P can never be at 10, because it must be at 6 or else at 12, so the two rock groups at 10 must be Q and T. Thus, because Q must be immediately after a Folk group, I inferred that there had to be at least one Folk group at 8. Same result as your approach but a step slower, I think. Well done!

Get the most out of your LSAT Prep Plus subscription.

Analyze and track your performance with our Testing and Analytics Package.